Đến nội dung

brianorosco

brianorosco

Đăng ký: 02-10-2014
Offline Đăng nhập: 15-11-2014 - 19:39
-----

#531743 Cho $a,b,c>0$ CMR: .....

Gửi bởi brianorosco trong 03-11-2014 - 23:24

Dùng cách khác cũng sẽ ra nhưng vs 1 con đường dài hơn @@ Vs lại nếu nói về S.O.S thì VN mình vẫn chưa đi sâu lắm. Mình cũng mới học S.O.S gần đây nên khi luyện tập thì đây là bài cơ bản nhất nên nói thế thôi ^^

Thật ra phương pháp S.O.S là phương pháp do người VN sáng tác đó bạn, nếu người VN chưa đi sâu thì nước nào đi sâu hả bạn???




#527961 $ab(a+b)+bc(b+c)+ca(c+a)$

Gửi bởi brianorosco trong 09-10-2014 - 19:47

1/Đặt $x=\frac{1}{a},y=\frac{1}{b},z=\frac{1}{c}$

Ta có giả thiết là $x+y+z+xy+yz+zx=6$

$3P=3x^2+3y^2+3z^2=(x^2+1)+(y^2+1)+(z^2+1)+2(x^2+y^2+z^2)-3\geq 2(x+y+z+xy+yz+zx)-3=9\Rightarrow P\geq 3$




#527800 Chứng minh rằng:$\frac{x^2}{(x-1)^2}+\frac...

Gửi bởi brianorosco trong 08-10-2014 - 18:51

Đặt:$a=\frac{x}{x-1},b=\frac{y}{y-1},c=\frac{z}{z-1}$

$(a-1)(b-1)(c-1)=\frac{1}{(x-1)(y-1)(z-1)}=abc \Rightarrow a+b+c=ab+bc+ca+1$

$a^{2}+b^{2}+c^{2}\geq 1\Leftrightarrow (a+b+c)^{2}+2\geq 2(a+b+c)+1\Leftrightarrow (a+b+c-1)^{2}$$\geq 0$




#527685 $\sum \sqrt[3]{\frac{a^6+b^6}{2}...

Gửi bởi brianorosco trong 07-10-2014 - 21:31

Ta chỉ cần chứng minh

$\frac{3(a^2+b^2)}{2}-2ab\geq \sqrt[3]{\frac{a^6+b^6}{2}} \Leftrightarrow (a-b)^2[ 9(a-b)^4+6a^2b^2+14a^4+14b^4-2a^3b-2ab^3]\geq 0$




#527679 CMR: $\left ( a-1 \right )\left (b-1 \right )\l...

Gửi bởi brianorosco trong 07-10-2014 - 21:13

3.Ta chứng minh bằng phép quy nạp toán học

Với $n=1,n=2$ bài toán hiển nhiên đúng.

Giả sử bài toán đúng với $k-2,k-1$ .Ta chứng minh nó đúng với $k$

Ta có:$(3-\sqrt{5})^{k}+(3+\sqrt{5})^{k}=[( 3-\sqrt{5})^{k-1}+( 3+\sqrt{5})^{k-1}] ( 3-\sqrt{5}+3+\sqrt{5} )-(3-\sqrt{5})(3+\sqrt{5})[( 3-\sqrt{5})^{k-2}+( 3+\sqrt{5})^{k-2}] =6[( 3-\sqrt{5})^{k-1}+( 3+\sqrt{5})^{k-1}]-4[( 3-\sqrt{5})^{k-2}+( 3+\sqrt{5})^{k-2}]$

CM xong!




#527526 $\boxed{\text{Chuyên Đề}}$ Bất đẳng thức - Cực trị

Gửi bởi brianorosco trong 06-10-2014 - 18:59

$(\frac{x^{2}}{y}+\frac{y^{2}}{z}+\frac{z^{2}}{x})(x+y+z)=(x^{2}+y^{2}+z^{2})+(\frac{x^{3}}{y}+\frac{y^{3}}{z}+\frac{z^{3}}{x})+(\frac{x^{2}z}{y}+\frac{y^{2}x}{z}+\frac{z^{2}y}{x})=(x^{2}+y^{2}+z^{2})+(\frac{x^{3}}{y}+xy+\frac{y^{3}}{z}+yz+\frac{z^{3}}{x}+zx+\frac{z^{3}}{x})+(\frac{x^{2}z}{y}+zy+\frac{y^{2}x}{z}+xz+\frac{z^{2}y}{x}+xy)-2(xy+yz+zx)\geq 3(x^{2}+y^{2}+z^{2})$




#527519 Chọn đội tuyển VMO Bắc Ninh 2014-2015

Gửi bởi brianorosco trong 06-10-2014 - 18:09

Câu 4:

a)$1=\frac{2015a^4+a^4}{2015a^4+a^4}$

Vậy tồn tại số đẹp.

Nếu $n$ là số đẹp

$\Rightarrow n=\frac{2015a^4+b^4}{2015c^4+d^4}\Rightarrow 16n=\frac{2015(2a)^4+(2b)^4}{2015c^4+d^4}$

Vậy 16n cũng là số đẹp.vậy tồn tại vô hạn số đẹp.

b)Ta cm bổ đề: Nếu $a^2+b^2\vdots p$ với p là số nguyên tố dạng $4k+3$ thì $a\vdots p$ và $b\vdots p$

Giả sử $a$ không chia hết cho p $\Rightarrow a^{p-1} \equiv 1$ (mod $p$)$\Rightarrow a^{4k+2}+b^{4k+2}\equiv 2$ (mod $p$)

Mà :$(a^{2})^{2k+1}+(b^{2})^{2k+1}\vdots a^{2}+b^{2}\vdots p$ (vô lí) $\Rightarrow$dpcm

Quay lại bài toán.Giả sử 2014 là số đẹp.

$\Rightarrow 2014=\frac{2015a^{4}+b^{4}}{2015c^{4}+d^{4}}$

Đặt:$(a,b,c,d)=m$$\Rightarrow a=mx,b=my,c=mz,d=mt,(x,y,z,t)=1$

$2015z^{4}+t^{4}=\frac{2015x^{4}+y^4}{2014}\Rightarrow x^{4}+y^{4}\vdots 1007\Rightarrow x\vdots 1007,y\vdots 1007\Rightarrow 2015z^{4}+t^{4}\vdots 1007\Rightarrow z\vdots 1007,t\vdots 1007$ (vô lí)

Vậy 2014 không phải là số đẹp




#527409 Kỹ thuật chọn điểm rơi trong Bất đẳng thức và cực trị

Gửi bởi brianorosco trong 05-10-2014 - 20:25

Lâu topic không hoạt động hôm nay mình xin post một số bài nhé.Mong các bạn học rồi thì cũng ôn lại

24,Cho $a,b>0;a+b\leq 1$.Tìm min:

a,$P=\frac{1}{a^2+b^2}+\frac{1}{2ab}$

b,$A=\frac{1}{1+a^2+b^2}+\frac{1}{2ab}$

 

 

a)$\frac{1}{a^2+b^2}+\frac{1}{2ab}\geq \frac{4}{\left ( a+b \right )^2}\geq 4$

b)$4ab\leq \left ( a+b \right )^2\leq 1$ (1)

$A=\frac{1}{1+\left ( a+b \right )^2-2ab}+\frac{1}{2ab}\geq\frac{1}{2-2ab}+\frac{1}{2ab}=\frac{1}{2}.(\frac{1}{1-ab}+\frac{1}{ab})$

Ta sẽ chứng minh:

$\frac{1}{1-ab}+\frac{1}{ab}\geq \frac{16}{3}\Leftrightarrow (4ab-1)(4ab-3)\leq 0$ ( đúng theo (1))




#527395 Tìm min $x^{2}+y^{2}+z^{2}$

Gửi bởi brianorosco trong 05-10-2014 - 19:53

$\left ( x-y \right )^2+\left ( y-z \right )^2+\left ( z-x \right )^2\geq 0\Rightarrow 2\left ( x^2+y^2+z^2 \right )\geq 2\left ( xy+yz+zx \right )\Rightarrow 3\left ( x^2+y^2+z^2 \right )\geq \left ( x+y+z \right )^2\Rightarrow x^2+y^2+z^2\geq \frac{2007^2}{3}$




#527132 Min ${{b + c} \over {{a^2}}...

Gửi bởi brianorosco trong 04-10-2014 - 16:49

Chứng minh được:$a^2 =b^2+c^2+bc$ và$b+c\leq \sqrt{\frac{4}{3}(b^2+c^2+bc)}=\frac{2a}{\sqrt{3}}$

VT$\geq$ $\left [ \frac{b+c}{a^2}+\frac{4}{3(b+c)}\right] +\frac{2}{3(b+c)}+\frac{5a\sqrt{3}}{36}\geq \frac{4}{a\sqrt{3}}+\frac{2}{3.\frac{2a}{\sqrt{3}}}+\frac{5a\sqrt{3}}{36}=5(\frac{1}{a\sqrt{3}}+\frac{a\sqrt{3}}{36})\geq \frac{5}{3}$

Dấu bằng xảy ra khi tam giác ABC cân tại A




#526964 Chứng minh:$\sum \frac{1}{{a^2 + bc}...

Gửi bởi brianorosco trong 02-10-2014 - 22:39

$a^2+bc\geq 2a\sqrt{bc} \Rightarrow \frac{1}{a^2+bc}\leq \frac{1}{2a\sqrt{bc}}=\frac{\sqrt{bc}}{2abc}$

Thiết lập các bdt tương tự,ta được:

VP$\leq \frac{\sqrt{bc}+\sqrt{ac}+\sqrt{ab}}{2abc}\leq \frac{a+b+c}{2abc}$ (dpcm)




#526962 $\sum \frac{9^{x}}{3^{x}+3^...

Gửi bởi brianorosco trong 02-10-2014 - 22:27

 

2.Cho x, y, z thỏa mãn 3-x +3-y +3-z =1

CMR$\frac{9^{x}}{3^{x}+3^{y+z}}+\frac{9^{y}}{3^{y}+3^{z+x}}+\frac{9^{z}}{3^{z}+3^{x+y}}\geq \frac{3^{x}+3^{y}+3^{z}}{4}$

Mình vừa học hàm số mũ -hàm số logarit nên dạng bài tập BĐT kiểu này mình thấy hơi khó làm :( .Mọi người vào giải giúp mình với!Thanks trước!

Đặt $a=3^{x}$, $b=3^{y}$, $c=3^{z}$ ,bài toán trở thành :

$ab+bc+ca=abc$ 

CM:$\frac{a^2}{a+bc}+\frac{a^2}{b+ac}+\frac{c^2}{c+ba}\geq \frac{a+b+c}{4}$

Ta có thể viết lại vế trái thành:

$\frac{a^3}{a^2+abc}+\frac{b^3}{b^2+abc}+\frac{c^3}{c^2+abc}=\frac{a^3}{(a+b)(a+c)}+\frac{b^3}{(b+c)(b+a)}+\frac{c^3}{(c+a)(c+b)}$

Và ta có đánh giá:

$\frac{a^3}{(a+b)(a+c)}+\frac{a+b}{8}+\frac{a+c}{8}\geq \frac{3a}{4}$

Cộng các bdt tương tự ta được đpcm.




#526932 $\boxed{\text{Chuyên Đề}}$ Bất đẳng thức - Cực trị

Gửi bởi brianorosco trong 02-10-2014 - 19:38

 

tôi còn bài này nữa nè mọi người :blink:

cho a,b,c >0 và a+b+c=3
cmr a/(ab+b^3) + b/(bc+c^3) + c/(ca+a^3)>= 3/2
p/s mọi nguoi ơi , làm thế nao để gõ dấu phân số vậy

 

$\frac{a}{ab+b^3}=\frac{1}{b}(\frac{a}{a+b^2})=\frac{1}{b}(1-\frac{b^2}{a+b^2})\geq \frac{1}{b}-\frac{b}{2b\sqrt{a}}=\frac{1}{b}-\frac{1}{2\sqrt{a}}$

Cộng hai bất đẳng thức tương tự, ta được:

VP $\geq \frac{1}{a}+\frac{1}{b}+\frac{1}{c}$$-(\frac{1}{2\sqrt{a}}+\frac{1}{2\sqrt{b}}+\frac{1}{2\sqrt{c}})$$= \frac{1}{4}\left [ (\frac{1}{\sqrt{a}}-1)^2+(\frac{1}{\sqrt{b}}-1)^2+(\frac{1}{\sqrt{c}}-1)^2\right ]+\frac{3}{4}(\frac{1}{a}+\frac{1}{b}+\frac{1}{c}-1)$ $\geq \frac{3}{4}\frac{9}{a+b+c}-\frac{3}{4}= \frac{3}{2}$